A: What is the perimeter of HOUSE?

perimeter of HOUSE =


(round to the nearest hundredth - two decimal places)


B: What is the length of OH?

(round to the nearest thousandth - three decimal places)


C: What is the midpoint of OU?


D: Which is longer, OH or EH?


E: What is the perimeter of the triangle HOU?


perimeter of triangle HOU


(round to the nearest hundredth - two decimal points)

A: What Is The Perimeter Of HOUSE?perimeter Of HOUSE =(round To The Nearest Hundredth - Two Decimal Places)B:

Answers

Answer 1

First calculate distances or lengths(Refer to attachment)

A:-

[tex]\\ \sf\longmapsto Perimeter=Sum\:of\:sides[/tex]

[tex]\\ \sf\longmapsto 5.3+9+11+9+9.7[/tex]

[tex]\\ \sf\longmapsto 15+29[/tex]

[tex]\\ \sf\longmapsto 44[/tex]

B:-

Refer to the attchment

C:-

[tex]\\ \sf\longmapsto \left(\dfrac{1+4}{2},\dfrac{1+6}{2}\right)[/tex]

[tex]\\ \sf\longmapsto \left(\dfrac{5}{2},\dfrac{7}{2}\right)[/tex]

D:-

EH>OH

E:-

[tex]\\ \sf\longmapsto Perimeter[/tex]

[tex]\\ \sf\longmapsto 9.7+5.3+9[/tex]

[tex]\\ \sf\longmapsto 24[/tex]

A: What Is The Perimeter Of HOUSE?perimeter Of HOUSE =(round To The Nearest Hundredth - Two Decimal Places)B:

Related Questions

need help with some khan academy stuff

Answers

Answer:

2.2 % per minute

35 minutes

Step-by-step explanation:

(0,23%) and (30, 89%)

We can find the slope

m = ( y2-y1)/(x2-x1)

    = (89-23)/(30-0)

   66/30

   =2.2 % per minute

To get to full change we need 100-23 = 77 %

77% * 1 minute/ 2.2 % =35 minutes

Pls answer this question! I will mark as brainliest!!!

Answers

Answer:

1/8

Step-by-step explanation:

There is a 1/2 probability for each of the three flips to result in the desired outcome. To get 3 desired outcomes in a row, we multiply each of the probabilities together

(1/2)³ = 1/8

Answer: it is D) 2/3

Someone please help!
-it is a 4 digit number.
-the vaule of the digit in the thousands place is 10 times greater than the value of the digit in the hundreds place.
-the least digit is in the tens place.
-the sun of the digits is 19. -the digit in the ones place is 4 more than the digit in the hundreds place.
-whats the mystery number?

Answers

Answer:

5539

Step-by-step explanation:

You should conclude that the number in the thousands place and the one in the 100s place are the same.

x * 1000 + x * 100 + y + x + 4

So you could sum the digits as

3x + y + 4 = 19   Subtract 4 from both sides

3x +  y = 15

x can be six           That means y = 1 which is OK

x can be five          That means y = 3 which is OK

Now we run into a little grief. Can x be 4? No

x cannot be 4          that would mean that y = 19 - 12 = 7 The tens digit is too big.

Come to think of it, x cannot be 6 because then the units digit will be 10 -- not possible.

So the answer is

1000*5 + 100*5 + 10*3 + 9

5539 is the answer.

Thanks for posting.

We want to find a 4-digit number that meets some given conditions. We will find two, which are:

4438 and 5509

A general 4-digit number can be written as:

a*1000 + b*100 + c*10 + d

Where a, b, c, and d are single-digit numbers.

a is the thousands digit

b is the hundreds digit

c is the tens digit

d is the units digit.

Here we know that:

"the value of the digit in the thousands place is 10 times greater than the value of the digit in the hundreds place."

So the first term is 10 times the second term, this means that:

(a*1000) = 10*(b*100)

a*1000 = b*1000

a = b

-the least (the smaller one) digit is in the tens place.

so:

c < a, b, d

- The sun of the digits is 19.

So:

a + b + c+ d = 19

-The digit in the ones place is 4 more than the digit in the hundreds place.

so:

d = b + 4

Now we can rewrite all of these conditions as:

b = ac < b, a, da + b + c + d = 19d = b + 4

Using the first one, we can replace all the b's by a's in the other conditions, so we have:

c <  a, d2a + c + d = 19d = a + 4

Now we can use the third equation and replace it in the second one to get:

2a + c + d = 19

2a + c + (a + 4) = 19

3a + c + 4 = 19

3a + c = 19 - 4

3a + c = 15

Now we need to remember that:

c < a.

So we can try with different values of a.

For example, if a = 3, then:

3*3 + c = 15

9 + c = 15

c = 15 - 9 = 6

here we have c > a, so we can discard this.

If a = 4, then:

3*4 + c = 15

12 + c = 15

c = 15 - 12 = 3

c < a

From this we can construct our number:

a = b = 4

c = 3

d = b + 4 = 4 + 4 = 8

Then the number is:

4438

If a = 5, then:

3*5 + c = 15

15 + c = 15

c = 15 - 15 = 0

c < a

So we also can have:

a = b = 5

c = 0

d = b + 4 = 9

Such that the number is:

5509

And if a is equal to or larger than 6 we have problems, because

3*6 + c = 15

18 + c =15

c = 15 -18 = -3

And c can't be a be a negative number.

So we can conclude that we found two 4-digit numbers that meet all the conditions, and these are:

4438 and 5509

If you want to learn more, you can read:

https://brainly.com/question/19902993

The median speed at which Dave drove to work was __________. 63 miles per hour 62 miles per hour 58.89 miles per hour 59 miles per hour

Answers

Step-by-step explanation:

Dave run with was 63 miles

Write the integer that represents the situation. °F represents the change The temperature was 15°F. It dropped so that the temperature was 0°F. in temperature. °F represents the change in temperature.​

Answers

Hi there!  

»»————- ★ ————-««

I believe your answer is:  

-15

»»————- ★ ————-««  

Here’s why:  

⸻⸻⸻⸻

The temperature dropped from 15°F to 0°F. The change was negative in value since it dropped to a lower temperature. The temperature dropped by 15 degrees, so we would fill in the blank with -15.

⸻⸻⸻⸻

»»————- ★ ————-««  

Hope this helps you. I apologize if it’s incorrect.  

Approximately 54% of mathematics students do their homework on time. In a class of 250 students, what is the mean, variance, and standard deviation if we assume normality and use the normal distribution as an approximation of the binomial distribution

Answers

Answer:

135

Step-by-step explanation:

250 x 54%=135

2. An electronic watch is sold for Rs 840 at a profit of Rs 90. Find its cost price and the profit per cent.

Answers

Answer:

750 CP AND PROFIT%=12%

Step-by-step explanation:

SP=rs 840

profit=90

CP=SP-profit

=840-90

=750

profit %=profit/CP*100%

=90/750*100%

=12%


At the zoo the polar bears are fed 0.1 bucket of fish a day.
The penguins are fed 0.83 that amount. How much of a
bucket are the penguins fed?

Answers

Answer:

0.083

Step-by-step explanation:

0.1 x 0.83 = 0.083

There are 0.083 bucket are the penguins fed.

What is Multiplication?

To multiply means to add a number to itself a particular number of times. Multiplication can be viewed as a process of repeated addition.

Given that;

At the zoo the polar bears are fed 0.1 bucket of fish a day.

And, The penguins are fed 0.83 that amount.

Now,

Since, At the zoo the polar bears are fed 0.1 bucket of fish a day.

And, The penguins are fed 0.83 that amount.

Hence, The bucket are the penguins fed is,

⇒ 0.83 × 0.1

⇒ 0.083

Learn more about the multiplication visit:

brainly.com/question/10873737

#SPJ2

Tom owns a landscaping business. He charges $40 for a yard cleanup, $50 to mow a lawn, and $75 to mulch a yard. On average, it takes Tom 25 minutes for a yard cleanup, 40 minutes to mow a lawn, and 2 hours to mulch a yard. Tom's clients are Mr. Hansen, Ms. Martinez, and Mrs. Johnson. . Mr. Hansen paid $125 for lawn services this week. . Tom spent more than an hour at Ms. Martinez' house this week. • Mrs. Johnson wrote Tom a check for $165 for the week. Tom made $405 from his three clients this week.

Answers

Answer:

Ms. Martinez =  Tom received $115, $75 to mulch the yard and $40 for a yard cleanup.

Step-by-step explanation:

Tom made $405 for the week from his three clients

Mr. Hansen paid Tom $125

Mrs. Johnson paid Tom $165

405-165-125=115

Ms. Martinez paid Tom $115, which meant that he would have cleaned the yard ($40) and mulch the yard ($75)

The unit amount and time for each service offered by Tom can be represented as equations. The conclusion from the given parameters are:

Mr. Hansen paid $125 to mow a lawn and to mulch a yard.Mrs. Johnson paid $165 for the three servicesMs. Martinez paid $115 for a yard cleanup and to mulch a yard.

Given that:

Cost

[tex]\$40 \to[/tex] Yard cleanup

[tex]\$50 \to[/tex] Mowing a lawn

[tex]\$75 \to[/tex] Mulching a yard

Time

[tex]2\ hrs \to[/tex] Yard cleanup

[tex]40\ mins \to[/tex] Mow a lawn

[tex]2\ hrs \to[/tex] Mulching a yard

[tex]Total\ Earnings = \$405[/tex]

From the question, we understand that:

[tex]Mr.\ Hansen = \$125[/tex]

Recall that:

[tex]\$50 \to[/tex] Mowing a lawn

[tex]\$75 \to[/tex] Mulching a yard

Add up:

[tex]\$50 + \$75 = 125[/tex]

This means that Mr. Hansen paid $125 to mow a lawn and to mulch a yard.

[tex]Mrs.\ Johnson= \$165[/tex]

Recall that:

[tex]\$40 \to[/tex] Yard cleanup

[tex]\$50 \to[/tex] Mowing a lawn

[tex]\$75 \to[/tex] Mulching a yard

Add up:

[tex]\$40 + \$50 + \$75 =\$165[/tex]

This means that Mrs. Johnson paid $165 for the three services

The amount paid by Ms. Martinez is calculated as follows:

[tex]Mr.\ Hansen + Mrs.\ Johnson + Ms.\ Martinez = Total\ Earnings[/tex]

Substitute known values

[tex]\$125 + \$165 + Ms.\ Martinez = \$405[/tex]

[tex]\$290 + Ms.\ Martinez = \$405[/tex]

Collect like terms

[tex]Ms.\ Martinez = \$405 - \$290[/tex]

[tex]Ms.\ Martinez = \$115[/tex]

This means that Ms. Martinez paid $115

Recall that:

[tex]\$40 \to[/tex] Yard cleanup

[tex]\$75 \to[/tex] Mulching a yard

Add up

[tex]\$40 +\$75 = \$115[/tex]

This means that Ms. Martinez paid $115 for a yard cleanup and to mulch a yard.

Read more about equations at:

https://brainly.com/question/2263981

447,077 is ____________________then 347,077.

Fill in the blank.​

Answers

447,077 is  greater than 347,077

447,077 greater than 347,077 by 100,000

447,077-347,077= 100,000

I can list more if you want! I hope I've helped!

Solve the given equation, enter your answer as "x= " and then your answer.

Example: x = 5

6=210

Answers

Answer:

×=204

Step-by-step explanation:

6=210

210-6

x=204

14 subtracted from three times a number is -53 find the number

Answers

Step-by-step explanation:

let the number

[tex] = x[/tex]

Then:

[tex](3x - 14) = 53 \\ 3x = 53 + 14 \\ 3x = 67 \\ x = 67 \div 3 = 22.33[/tex]

Answer:

[tex]number = x \\ 3x - 14 = - 53 \\ 3x = - 53 + 14 \\ 3x = - 39 \\ x = \frac{ - 39}{3} \\ x = - 13 \\ thank \: you[/tex]

Asap right now brainliest for the correct one

Answers

Answer:

Option B

Step-by-step explanation:

The next step is the result of calculating the square roots:

[tex]4^{1/2} = 2[/tex]

and

[tex]36^{1/2} = 6[/tex]

This will make the step:

2 · [tex]3^{1/2}[/tex] + 6 · [tex]3^{1/2}[/tex] = 8 · [tex]3^{1/2}[/tex]

Correct choice is B

eksponent

[tex]\sqrt[n]{x^n} = x[/tex][tex]\sqrt{x} = x^{1/2}[/tex]

[tex] = 4^{ \frac{1}{2} } .3 ^{ \frac{1}{2} } + \sqrt{36} \times 3^ \frac{1}{2} [/tex]

[tex] = \sqrt{4} \times 3 {}^{ \frac{1}{2} } + 6 \times 3 {}^{ \frac{1}{2} } [/tex]

[tex] = 2 \times 2 + 6 \times 3 {}^{ \frac{1}{2} } [/tex]

[tex] = 4 + 6 \times \sqrt{3} [/tex]

[tex] = 4 + 6 \sqrt{3} [/tex]

Arundel Corporation has 7,000 shares of 6% $100 par cumulative preferred stock outstanding. Arundel paid all preferred dividends due for the year 2019 but paid no dividends in 2020. What amount will Arundel need to pay preferred shareholders in 2021 if they wish to pay a dividend to common shareholders?

Answers

The amount will Arundel need to pay preferred shareholders in 2021 if they wish to pay a dividend to common shareholders is 84,000

Using this formula

2021 preferred shareholders(per year)=Per dividend * Dividend rate

Let plug in the formula

2021 preferred shareholders(per year)=[(7,000*$100)*6%]

2021 preferred shareholders(per year)=($700,000*6%)*2

2021 preferred shareholders(per year)=42,000*2

2021 preferred shareholders(per year)=84,000

Inconclusion The amount will Arundel need to pay preferred shareholders in 2021 if they wish to pay a dividend to common shareholders is 84,000

Learn more about preferred shareholders here:

https://brainly.com/question/3518273

Solve the equation. x + (–6) = –9

Answers

Answer:

-3

Step-by-step explanation:

9514 1404 393

Answer:

  x = -3

Step-by-step explanation:

Undo the addition of -6 by adding the opposite of that to both sides of the equation.

  x + (-6) +(+6) = (-9) +(+6)

  x -6 +6 = -9 +6

  x = -3

Use a number line or place value to round each number to the place of the underlined digit

Answers

you didn't put a picture in,

9 gal 2 qt to quarts

Answers

Answer:

38 us liquid quarts

Step-by-step explanation:

Pls Help…

11 Alicia buys a car that costs $26610, and she makes a $3140 down payment on it. How much
does she still owe on the car? Give units.

Answers

Answer:

$23,470

Step-by-step explanation:

A down payment is how much you pay upfront for a car. So, how much she still owes will be the difference between the total cost and the down payment. Therefore, subtract $3,140 from $26,610; 26610-3140=23470. Then, since you are talking about money the units are dollars, $.

Simplify the following expression completely, where x

Answers

Step-by-step explanation:

[tex]x \sqrt{5x {y}^{4} } + \sqrt{405 {x}^{3} y {}^{4} } - \sqrt{80 {x}^{3} {y}^{4} } [/tex]

[tex]x \sqrt{5xy {}^{4} } + 9 x\sqrt{5xy {}^{4} } - 4x \sqrt{5xy {}^{4} } [/tex]

[tex]6x \sqrt{5xy {}^{4} } [/tex]

Answer:

[tex]x\sqrt{5xy^4} + \sqrt{405x^3y^4} -\sqrt{80x^3y^4} =[/tex]

[tex]xy^2\sqrt{5x} + 9xy^2\sqrt{5x} -4xy^2\sqrt{5x} =[/tex]

[tex]xy^2\sqrt{5x} (1 + 9 - 4) =[/tex]

[tex]6xy^2\sqrt{5x}[/tex]

In math how can you tell if something is a function.

Answers

Answer:

if the line dosent have the same x's

Step-by-step explanation:

35 POINTS, i dont understand this question!!!! can someone give me a brief yet useful explanation??

Answers

The answer:
surveying a random sample of 25 students attending a school football game

Don't forget put heart ♥️

3s(4 + 6)2 is an example of

Answers

Answer:

A. An Algebraic Expression

Step-by-step explanation:

Since we have no equal sign in the problem, we know that it is an expression. An equation has an equal sign and another value so we can simplify to find the value of whatever variable we are working with. We also know that it is not anything numerical because we are working with variables. With this information we will see that the correct answer is A.

Best of Luck!

Answer:

options A an algebraic expression

Step-by-step explanation:

please mark me brainleast

Anyone who knows this mathematic question will get the brainliest

Answers

The third one is 510 not show in the pic here

Which expression does this power represent?

109


10 × 10 × 10 × 10 × 10 × 10 × 10 × 10 × 10

9 × 9 × 9 × 9 × 9 × 9 × 9 × 9 × 9 × 9

10 × 9 × 10 × 9 × 10 × 9 × 10 × 9 × 10 × 9 × 10 × 9 × 10 × 9 × 10 × 9 × 10 × 9

10 × 9

Answers

9514 1404 393

Answer:

  (a)  10 × 10 × 10 × 10 × 10 × 10 × 10 × 10 × 10

Step-by-step explanation:

You may recall that we use a multiplier to signify repeated addition:

  x + x + x = 3x

In a similar way, we use an exponent to signify repeated multiplication:

  10 × 10 × 10 = 10³

__

The exponent 9 in 10⁹ means the factor 10 appears 9 times in the product:

  10 × 10 × 10 × 10 × 10 × 10 × 10 × 10 × 10 = 10⁹

Answer:

a  10 × 10 × 10 × 10 × 10 × 10 × 10 × 10 × 10

Step-by-step explanation:

12. Solve x^2 + 6x - 16 = 0 by completing the square.


Please help and show work

Answers

Answer:

x is 2 and -8

Step-by-step explanation:

[tex] {x}^{2} + 6x - 16 = 0[/tex]

general equation

[tex] {ax}^{2} + (sum)x + product = 0[/tex]

sum is 6, product is -16

for completing squares,

first divide the sum by 2:

[tex] = \frac{6}{2} = 3[/tex]

add the square of the result on (x² + 6x) and subtract it from the product:

[tex] ( {x}^{2} + 6x + {3}^{2} ) - 16 - {3}^{2} = 0 \\ {(x + 3)}^{2} - 25 = 0 \\ {(x + 3)}^{2} = 25[/tex]

take square root:

[tex] \sqrt{ {(x + 3)}^{2} } = \sqrt{25 } \\ x + 3 = ±5 \\ x = ±5 - 3[/tex]

x is either: 5-3 or -5-3

[tex]x = 2 \: \: and \: - 8[/tex]

if x^3+ax^2+bx=6 is exactly divisible by (x-1), prove that a+b=5.​

Answers

Step-by-step explanation:

x-1=0

x=1

subtitute value of x

p(x)=x^3+ax^2+bx

p(1)=(1)^3+a(1)^2+b(1)

p(1)=1+a+b

as p(x) =6. (given)

1+a+b=6

a+b=6-1

a+b=5

Which expression is equivalent to 10° ?
o 10 - 10 - 10 - 10 - 10 - 10
O 6.6.6.6.6.6.6.6.6.6
0 10. 10. 10. 10.10
o I don't know.
Hi

Answers

Answer:

The first one: 10*10*10*10*10*10

Step-by-step explanation:

This is because the power of a number is how many times you multiply it by itself (example: [tex]2^{4}[/tex] = 2*2*2*2 because the power is 4 which means you multiply the base by itself 4 times, in this case the base is 2)

Answer:

A. 10 · 10 · 10 · 10 · 10 · 10

Step-by-step explanation:

[tex]10^6[/tex] is just 10 times itself, 6 times.

16. If there are 15 girls and 45 boys in a class. Then, find the percentage of boys and girls.​

Answers

Answer:

25% and 75%

Step-by-step explanation:

15+45=60

girl =  [tex]\frac{15}{60} *100[/tex] = 25%

boy = [tex]\frac{45}{60} *100[/tex] = 75%


Shellie buys a loaf of bread fo $2.99, a gallon of milk for $3.49, and a pack of
lunchmeat for $5.99. The sales tax is 8%. How much will Shellie's total bill be,
including the sales tax? Round your answer to the nearest cent.

Answers

Answer:

$13.47

Step-by-step explanation:

2.99 + 3.49 +5.99 + 12.47.

12.47 x 0.08 = 1.

12.47  + 1 = 13.47

Anyone know the answer with explanation please I would appreciate it!

Answers

Answer:

[tex]\displaystyle QR = 12[/tex]

Step-by-step explanation:

First, it's never a bad idea to draw the line and see what it looks like! This is shown below (not to scale).

We are given that PS = 18 and PR = 15, and we want to determine QR.

PS is the sum of PR and RS:

[tex]\displaystyle PS = PR + RS[/tex]

Substitute:

[tex]\displaystyle (18) = (15) + RS[/tex]

Solve for RS

[tex]\displaystyle RS = 3[/tex]

Since RS ≅ PQ:

[tex]\displaystyle RS = PQ = 3[/tex]

PS is also the sum of PQ, QR, and RS. Hence:

[tex]PS = PQ + QR + RS[/tex]

Since PS = 18 and RS = PQ = 3:

[tex](18) = (3) + QR + (3)[/tex]

Solve for QR:

[tex]QR = 12[/tex]

In conclusion, QR measures 12 units.

Other Questions
what is the solution of linear equation of cosx(e^2y-y)=e^y sin2x ASAP Write the following phrase as a variable expression. Use x to represent "a number".the sum of a number and one The translation is ____ What is the value of x in the equation 0.7x 1.4 = -3.5?O -7 37 find product of :(2x-y)(2x+3y). 11(5) - 11(4) I really need helpon this Can you locate your city and state on the map drawn by Ptolemy?A. No it will be too old of a map to be accurateB. No it would be missing parts of the United StatesC. No it would be of the stars in the skyD. Yes I could find my city and state on map drawn by PTOLEMY (x + 3)(x-8)Multiply and simplify To what is George referring to when he say you say father did it MARKING BRAINLIEST !! using the distance formula falculate the perimeter of the triangle belowa. 27.9 units b. 24.3 units c. 25.7 units d. 28.2 units According to the diagram above how long is the unknown piece Briefly describe any one artists background, style, techniques, and famous works. Does anyone has the answers for determining central idea clean these choppers ? Its 7th grade . find the value proportion :x : 35:98:60 if f(x)=3x+8/2x-Aand f(0)=2.what is the value of a? 7) Identify0 asimile used inthe text Explainthe two things thatbeing comparedthat are-)Ford why are Polar bears are much larger than American black bears 10. State the divisibility rules for 4 and 8 and check if the number 684572 is divisible by 4 and 8. cuanto es el tiempo mximo para estar en mi realidad deseada! What is an example of positive feedback?A)A herd of buffalo running away after one of them spots a lionB)A pressure cooker valve opening to reduce the high pressure insideC) A computer screen dimming or when the battery gets lowD) A thermostat switching on an air conditioner when the temperature gets too hot helpppp i dont understannnnd